Can the Partial Sum of the Cosine Telescoping Series be Negative?

Click For Summary
SUMMARY

The discussion centers on the partial sum of the cosine telescoping series defined by the expression Ʃ (cos(1/(n)^2) - cos(1/(n+1)^2)). It is established that the nth partial sum results in cos(1) - cos(1/(n+1)^2). As n approaches infinity, this limit evaluates to cos(1) - 1, which is negative. The use of Wolfram Alpha confirms this result, demonstrating that partial sums can indeed be negative.

PREREQUISITES
  • Understanding of telescoping series
  • Familiarity with limits in calculus
  • Basic knowledge of trigonometric functions, specifically cosine
  • Experience with computational tools like Wolfram Alpha
NEXT STEPS
  • Explore the properties of telescoping series in greater detail
  • Study the behavior of limits involving trigonometric functions
  • Learn how to use Wolfram Alpha for series and limits
  • Investigate other examples of series that yield negative partial sums
USEFUL FOR

Students studying calculus, mathematicians interested in series convergence, and anyone exploring the properties of trigonometric functions in series.

Jbreezy
Messages
582
Reaction score
0

Homework Statement



If you sum this
from one to infinity.
Ʃ (cos(1/(n)^2 - cos(1/(n+1)^2)


Homework Equations





The Attempt at a Solution



Ʃ (cos(1/(n-1)^2 - cos(1/(n+1)^2)
This is telescoping if you work that out for the partial nth partial sum you get

cos(1) - cos(1/(n+1)^2) if you take the limit you get cos(1) -1 which is negative. If you punch it in on wolfram you get the same thing. Can the partial sum be negative ?
 
Physics news on Phys.org
Jbreezy said:

Homework Statement



If you sum this
from one to infinity.
Ʃ (cos(1/(n)^2 - cos(1/(n+1)^2)


Homework Equations





The Attempt at a Solution



Ʃ (cos(1/(n-1)^2 - cos(1/(n+1)^2)
This is telescoping if you work that out for the partial nth partial sum you get

cos(1) - cos(1/(n+1)^2) if you take the limit you get cos(1) -1 which is negative. If you punch it in on wolfram you get the same thing. Can the partial sum be negative ?

Yes.
 
Can you elaborate please.
 
What's to elaborate? Partial sums can be negative. You are looking at one. Plug in some values of n and see.
 
Question: A clock's minute hand has length 4 and its hour hand has length 3. What is the distance between the tips at the moment when it is increasing most rapidly?(Putnam Exam Question) Answer: Making assumption that both the hands moves at constant angular velocities, the answer is ## \sqrt{7} .## But don't you think this assumption is somewhat doubtful and wrong?

Similar threads

  • · Replies 1 ·
Replies
1
Views
2K
Replies
6
Views
2K
Replies
2
Views
5K
  • · Replies 3 ·
Replies
3
Views
2K
  • · Replies 1 ·
Replies
1
Views
1K
  • · Replies 4 ·
Replies
4
Views
3K
  • · Replies 2 ·
Replies
2
Views
1K
Replies
6
Views
1K
  • · Replies 12 ·
Replies
12
Views
3K
  • · Replies 2 ·
Replies
2
Views
2K